Art of Problem Solving
During AMC 10A/12A testing, the AoPS Wiki is in read-only mode and no edits can be made.

2008 AIME II Problems/Problem 9: Difference between revisions

Blackhawk (talk | contribs)
No edit summary
Blackhawk (talk | contribs)
No edit summary
Line 12: Line 12:
<center><math>5\sqrt {2} + 5(\sqrt {2} + 1) \approx 19.1 \Longrightarrow \boxed{019}</math></center>
<center><math>5\sqrt {2} + 5(\sqrt {2} + 1) \approx 19.1 \Longrightarrow \boxed{019}</math></center>


If you're curious, [url="https://www.desmos.com/calculator/febtiheosz"]the points[/url] do eventually form an octagon and repeat. Seems counterintuitive, but believe it or not, it happens.
If you're curious, the points do eventually form an octagon and repeat. Seems counterintuitive, but believe it or not, it happens.


https://www.desmos.com/calculator/febtiheosz
=== Solution 2 ===
=== Solution 2 ===
Let the particle's position be represented by a complex number. Recall that multiplying a number by where a is cis<math>\left( \theta \right)</math>. rotates the object in the complex plane by <math>\theta</math> counterclockwise. Therefore, applying the rotation and shifting the coordinates by 10 in the positive x direction in the complex plane results to
Let the particle's position be represented by a complex number. Recall that multiplying a number by where a is cis<math>\left( \theta \right)</math>. rotates the object in the complex plane by <math>\theta</math> counterclockwise. Therefore, applying the rotation and shifting the coordinates by 10 in the positive x direction in the complex plane results to

Revision as of 17:46, 28 October 2017

Problem

A particle is located on the coordinate plane at $(5,0)$. Define a move for the particle as a counterclockwise rotation of $\pi/4$ radians about the origin followed by a translation of $10$ units in the positive $x$-direction. Given that the particle's position after $150$ moves is $(p,q)$, find the greatest integer less than or equal to $|p| + |q|$.

Solution

Solution 1

Let P(x, y) be the position of the particle on the xy-plane, r be the length OP where O is the origin, and $\theta$ be the inclination of OP to the x-axis. If (x', y') is the position of the particle after a move from P, then $x'=rcos(\pi/4+\theta)+10 = \sqrt{2}(x - y)/2 + 10$ and $y' = rsin(\pi/4+\theta) = \sqrt{2}(x + y)/2$. Let $(x_n, y_n)$ be the position of the particle after the nth move, where $x_0 = 5$ and $y_0 = 0$. Then $x_{n+1} + y_{n+1} =  \sqrt{2}x_n+10$, $x_{n+1} - y_{n+1} = -\sqrt{2}y_n+10$. This implies $x_{n+2} = -y_n + 5\sqrt{2}+ 10$, $y_{n+2}=x_n + 5\sqrt{2}$. Substituting $x_0 = 5$ and $y_0 = 0$, we have $x_8 = 5$ and $y_8 = 0$ again for the first time. Thus, $p = x_{150} = x_6 = -5\sqrt{2}$ and $q = y_{150} = y_6 = 5 + 5\sqrt{2}$. Hence, the final answer is

$5\sqrt {2} + 5(\sqrt {2} + 1) \approx 19.1 \Longrightarrow \boxed{019}$

If you're curious, the points do eventually form an octagon and repeat. Seems counterintuitive, but believe it or not, it happens.

https://www.desmos.com/calculator/febtiheosz

Solution 2

Let the particle's position be represented by a complex number. Recall that multiplying a number by where a is cis$\left( \theta \right)$. rotates the object in the complex plane by $\theta$ counterclockwise. Therefore, applying the rotation and shifting the coordinates by 10 in the positive x direction in the complex plane results to

$a_{150} = (((5a + 10)a + 10)a + 10 \ldots) = 5a^{150} + 10 a^{149} + 10a^{148}+ \ldots + 10$

where a is cis$\left( \theta \right)$. By De-Moivre's theorem, $\left(cis( \theta \right)^n )$=cis$\left(n \theta \right)$. Therefore,

$10(a^{150} + \ldots + 1) = 10(1 + a + \ldots + a^6) = - 10(a^7) = - 10(\frac{ \sqrt {2} }{2} - \frac{i\sqrt {2}} {2})$

Furthermore, $5a^{150} = - 5i$. Thus, the final answer is

$5\sqrt {2} + 5(\sqrt {2} + 1) \approx 19.1 \Longrightarrow \boxed{019}$

See also

2008 AIME II (ProblemsAnswer KeyResources)
Preceded by
Problem 8
Followed by
Problem 10
1 2 3 4 5 6 7 8 9 10 11 12 13 14 15
All AIME Problems and Solutions

These problems are copyrighted © by the Mathematical Association of America, as part of the American Mathematics Competitions. Error creating thumbnail: Unable to save thumbnail to destination